Difference between revisions of "1999 AHSME Problems/Problem 25"

(+Solution 2)
(One intermediate revision by one other user not shown)
Line 17: Line 17:
 
<cmath>\frac{5 \cdot 6!-2}{7} = 514 = 360a_2 + 120a_3 + 30a_4 + 6a_5 + a_6.</cmath>  
 
<cmath>\frac{5 \cdot 6!-2}{7} = 514 = 360a_2 + 120a_3 + 30a_4 + 6a_5 + a_6.</cmath>  
  
We then repeat this procedure <math>\pmod{6}</math>, from which it follows that <math>a_6 \equiv 514 \equiv 4 \pmod{6}</math>, and so forth. Continuing, we find the unique solution to be <math>(a_2, a_3, a_4, a_5, a_6, a_7) = (1,1,1,0,4,2)</math> (uniqueness is assured by the [[Division Theorem]]). The answer is <math>9 \Longrightarrow \mathrm{(B)}</math>.
+
We then repeat this procedure <math>\pmod{6}</math>, from which it follows that <math>a_6 \equiv 514 \equiv 4 \pmod{6}</math>, and so forth. Continuing, we find the unique solution to be <math>(a_2, a_3, a_4, a_5, a_6, a_7) = (1,1,1,0,4,2)</math> (uniqueness is assured by the [[Division Theorem]]). The answer is <math>9 \Longrightarrow \boxed{\mathrm{(B)}}</math>.
  
 +
== Solution 2 ==
 +
We start by multiplying both sides by <math>7!</math>, and we get:
 +
<cmath>3600=2520a_2+840a_3+210a_4+42a_5+7a_6+a_7</cmath>
 +
After doing some guess and check, we find that the answer is <math>\boxed{\textbf{(B)~9}}</math>.
 +
 +
~aopspandy
 
== See also ==
 
== See also ==
 
{{AHSME box|year=1999|num-b=24|num-a=26}}
 
{{AHSME box|year=1999|num-b=24|num-a=26}}

Revision as of 22:59, 20 May 2021

Problem

There are unique integers $a_{2},a_{3},a_{4},a_{5},a_{6},a_{7}$ such that

\[\frac {5}{7} = \frac {a_{2}}{2!} + \frac {a_{3}}{3!} + \frac {a_{4}}{4!} + \frac {a_{5}}{5!} + \frac {a_{6}}{6!} + \frac {a_{7}}{7!}\]

where $0\leq a_{i} < i$ for $i = 2,3,\ldots,7$. Find $a_{2} + a_{3} + a_{4} + a_{5} + a_{6} + a_{7}$.

$\textrm{(A)} \ 8 \qquad \textrm{(B)} \ 9 \qquad \textrm{(C)} \ 10 \qquad \textrm{(D)} \ 11 \qquad \textrm{(E)} \ 12$

Solution

Multiply out the $7!$ to get

\[5 \cdot 6! = (3 \cdot 4 \cdots 7)a_2 + (4 \cdots 7)a_3 + (5 \cdot 6 \cdot 7)a_4 + 42a_5 + 7a_6 + a_7 .\]

By Wilson's Theorem (or by straightforward division), $a_7 + 7(a_6 + 6a_5 + \cdots) \equiv 5 \cdot 6! \equiv -5 \equiv 2 \pmod{7}$, so $a_7 = 2$. Then we move $a_7$ to the left and divide through by $7$ to obtain

\[\frac{5 \cdot 6!-2}{7} = 514 = 360a_2 + 120a_3 + 30a_4 + 6a_5 + a_6.\]

We then repeat this procedure $\pmod{6}$, from which it follows that $a_6 \equiv 514 \equiv 4 \pmod{6}$, and so forth. Continuing, we find the unique solution to be $(a_2, a_3, a_4, a_5, a_6, a_7) = (1,1,1,0,4,2)$ (uniqueness is assured by the Division Theorem). The answer is $9 \Longrightarrow \boxed{\mathrm{(B)}}$.

Solution 2

We start by multiplying both sides by $7!$, and we get: \[3600=2520a_2+840a_3+210a_4+42a_5+7a_6+a_7\] After doing some guess and check, we find that the answer is $\boxed{\textbf{(B)~9}}$.

~aopspandy

See also

1999 AHSME (ProblemsAnswer KeyResources)
Preceded by
Problem 24
Followed by
Problem 26
1 2 3 4 5 6 7 8 9 10 11 12 13 14 15 16 17 18 19 20 21 22 23 24 25 26 27 28 29 30
All AHSME Problems and Solutions

The problems on this page are copyrighted by the Mathematical Association of America's American Mathematics Competitions. AMC logo.png